The position that punishment should be proportional to how serious the offense is but that repeat offenders should re...

wills on May 28, 2019

Question explanation

Could you break down this question? Is the conclusion the first or the last sentence? I couldn't figure this out so many of the answer choices seemed to be applicable to the statement.

Replies
Create a free account to read and take part in forum discussions.

Already have an account? log in

Ravi on May 30, 2019

@wills,

Happy to help.

The conclusion of the stimulus is the first sentence. If you're stuck
and can't figure out the conclusion between two sentences, a good
trick is to put the word "because" in between the sentences and then
see which way makes more sense.

The position that punishment should be proportional to how serious the
offense is but that repeat offenders should receive harsher
punishments than first-time offenders is unsustainable BECAUSE this
would make determining the seriousness of an offense so difficult that
it would be impossible to apply the proportionality principle.

Here, the first sentence is before "because" and it makes sense
because the last sentence provides support.

This would make determining the seriousness of an offense so difficult
that it would be impossible to apply the proportionality principle
BECAUSE the position that punishment should be proportional to how
serious the offense is but that repeat offenders should receive
harsher punishments than first-time offenders is unsustainable.

Here, the last sentence is before "because," but it doesn't make
sense. This is how we know that the first sentence is the main
conclusion; it's what the whole stimulus is working to support.

The question asks, "The statement that considerations as remote as
what an offender did years ago are relevant to the seriousness of an
offense plays which one of the following roles in the argument?"

Now that we know the conclusion, let's look at the phrase in question.
We see that it's discussing an indefensible consequence of the
position that the overall conclusion is rejecting, which leads us to
(D).

Hope this helps. Let us know if you have any more questions!

hspring on April 24, 2020

I understood the conclusion but got the question wrong. I picked A . Can you please outline this further? I'm struggling with the vagueness of the answers I think.